1answer.
Ask question
Login Signup
Ask question
All categories
  • English
  • Mathematics
  • Social Studies
  • Business
  • History
  • Health
  • Geography
  • Biology
  • Physics
  • Chemistry
  • Computers and Technology
  • Arts
  • World Languages
  • Spanish
  • French
  • German
  • Advanced Placement (AP)
  • SAT
  • Medicine
  • Law
  • Engineering
marshall27 [118]
3 years ago
7

Mr. Winking is purchasing a car and needs to finance $24,000 from the bank with an annual percentage rate (APR)

Mathematics
1 answer:
babymother [125]3 years ago
5 0

Answer:

<em>The monthly payment is $450.71</em>

Step-by-step explanation:

<u>Financial Computing </u>

Given the loan amount A, the loan term t, and the APR (annual percentage rate), the montly payment is computed as

\displaystyle P=\frac{A}{f}

where f is

\displaystyle f=\frac{1-(1+i)^{-n}}{i}

The provided data is

\displaystyle A=24,000

\displaystyle r=4.8\%

\displaystyle t=5\ years

Since the payments will be made monthly, the values of n and i are:

\displaystyle i=\frac{4.8}{100\times12}=0.004

\displaystyle n=5\times12=60\ months

Calculating f:

\displaystyle f=\frac{1-(1+i)^{-n}}{i}

\displaystyle f=\frac{1-(1+0.004)^{-60}}{0.004}

\displaystyle f=53.25

Now for the payments:

\displaystyle P=\frac{24.000}{53.25}=450.71

\boxed{\displaystyle P=\$450.71}

You might be interested in
Ln a+3 + ln a-3 = ln 16
Lilit [14]
\bf log_{{  a}}(xy)\implies log_{{  a}}(x)+log_{{  a}}(y)\\\\&#10;and\qquad \textit{difference of squares}&#10;\\ \quad \\&#10;(a-b)(a+b) = a^2-b^2\qquad \qquad &#10;a^2-b^2 = (a-b)(a+b)\\\\&#10;-----------------------------\\\\&#10;ln(a+3)+ln(a-3)=ln(16)\implies ln[(a+3)(a-3)]=ln(16)&#10;\\\\\\&#10;ln[a^2-3^2]=ln(16)\impliedby \textit{removing ln() from both sides}&#10;\\\\\\&#10;a^2-9=16\implies a^2=16+9\implies a^2=25&#10;\\\\\\&#10;a=\pm\sqrt{25}\implies a=\pm 5
7 0
3 years ago
The distribution of the amount of money spent by first-time gamblers at a major casino in las vegas is approximately normal in s
Lostsunrise [7]

Solution: We are given:

\mu=600, \sigma =120

We need to find the z value corresponding to probability 0.84, in order to find the how much money almost 84% of gamblers spent at casino.

Using the standard normal table, we have:

z(0.85) = 0.9945

Now we will use the z score formula to find the required amount:

z=\frac{x-\mu}{\sigma}

0.9945=\frac{x-600}{120}

0.9945 \times 120 = x - 600

119.34 = x - 600

x = 600 + 119.34        

x = 719.34

x = 720 approximately

Therefore, almost 84% of gamblers spent more than $720 amount of money at this casino.

5 0
3 years ago
Marianne collects coffee mugs from places she visits when she goes on vacation. She displays her 87 coffee mugs over her cabinet
lozanna [386]
So the probability of it being California a
4 0
4 years ago
Divide 29 into two parts so that the sun of the squares of the parts is 425.Find the value of each Part.
Taya2010 [7]

Answer:

13 and 16

Step-by-step explanation:

let the 2 parts be x and y, then

x + y = 29 → (1) and

x² + y² = 425 → (2)

From (1) → x = 29 - y → (3)

substitute x = 29 - y into (2)

(29 - y)² + y² = 425 ( expand factor )

841 - 58y + y² + y² = 425 ( rearrange into standard form )

2y² - 58y + 416 = 0 ← in standard quadratic form

divide all terms by 2

y² - 29y + 208 = 0

Consider the factors of 208 which sum to - 29

These are - 13 and - 16, hence

(y - 13)(y - 16) = 0

equate each factor to zero and solve for y

y - 13 = 0 ⇒ y = 13

y - 16 = 0 ⇒ y = 16

substitute these values into (3)

x = 29 - 13 = 16 and x = 29 - 16 = 13

The 2 parts are 13 and 16


4 0
3 years ago
Read 2 more answers
The perimeter of a rectangle is 68 ft. Find the dimensions of the rectangle if the ratio of the length to the width is 9 : 8. Wh
Kipish [7]
I think the best answer is d since 18x + 16x
7 0
3 years ago
Other questions:
  • Please answer this correctly
    5·2 answers
  • If shelia earned 36 points for the ducks, how many points did she earn for each frog?
    10·2 answers
  • What is the value of x to the nearest tenth the figure is not you scale x=2.1 x=3.4 x=9.6 x=13.1
    15·2 answers
  • What side is adjacent to angle Y?<br> XZ<br> ZY<br> YX<br> None of the above
    5·1 answer
  • Pleaseeee help meeeeeee
    13·1 answer
  • ILL GIVE U BRAINLIST!!
    13·1 answer
  • Find the first three terms of the sequence below. T n = n 2 + 3 n + 4
    13·1 answer
  • Solve the inequalities by graphing. ldentify the graph that shows the following equations. PLS PLS HELP ME
    11·1 answer
  • Prices are going up in the ratio 6:5 Write a 60% decrease as a ratio.
    10·1 answer
  • HELP ASAP I NEED THIS ANSWERED AS SOON AS POSSIBLE
    9·1 answer
Add answer
Login
Not registered? Fast signup
Signup
Login Signup
Ask question!